-
Notifications
You must be signed in to change notification settings - Fork 0
/
hw01.tex
238 lines (206 loc) · 13 KB
/
hw01.tex
1
2
3
4
5
6
7
8
9
10
11
12
13
14
15
16
17
18
19
20
21
22
23
24
25
26
27
28
29
30
31
32
33
34
35
36
37
38
39
40
41
42
43
44
45
46
47
48
49
50
51
52
53
54
55
56
57
58
59
60
61
62
63
64
65
66
67
68
69
70
71
72
73
74
75
76
77
78
79
80
81
82
83
84
85
86
87
88
89
90
91
92
93
94
95
96
97
98
99
100
101
102
103
104
105
106
107
108
109
110
111
112
113
114
115
116
117
118
119
120
121
122
123
124
125
126
127
128
129
130
131
132
133
134
135
136
137
138
139
140
141
142
143
144
145
146
147
148
149
150
151
152
153
154
155
156
157
158
159
160
161
162
163
164
165
166
167
168
169
170
171
172
173
174
175
176
177
178
179
180
181
182
183
184
185
186
187
188
189
190
191
192
193
194
195
196
197
198
199
200
201
202
203
204
205
206
207
208
209
210
211
212
213
214
215
216
217
218
219
220
221
222
223
224
225
226
227
228
229
230
231
232
233
234
235
236
237
238
\documentclass[
pages,
boxes,
color=RoyalPurple
]{homework}
\usepackage{macros}
\usepackage[footnote,nolist,smaller]{acronym}
\usepackage{cleveref}
\input{config}
\hwnum{1}
\duedate{Mon, Feb 22, 2020 11:59 PM}
\colorlet{Ppink}{WildStrawberry!20}
\newtcbox{\mafbox}[1][]{on line, math upper,
boxsep=4pt, left=0pt,right=0pt,top=0pt,bottom=0pt,
colframe=white,colback=Ppink,
highlight math style={enhanced}}
\makeatletter
\numberwithin{tcb@cnt@prob}{section}
\makeatother
\begin{document}
\begin{acronym}
\acro{PVM}{Projection-Valued Measure}
\acro{POVM}{Positive Operator-Valued Measure}
\end{acronym}
\setcounter{section}{2}
\problemnumber{2}
\begin{problem}
Show that the reduced state obtained via partial trace is a density operator, i.e., a non-negative operator satisfying $\tr\hat{\rho}_A = 1$.
\end{problem}
\begin{solution}
Take $\hat{\rho}_{AB}$ to be a density operator on Hilbert space $\mathcal{H}_{AB}$. In a (tensor product) basis this looks like
\begin{equation*}
\hat{\rho}_{AB} = \sum_{i,j,k,\ell}p_{ijk\ell}\ket{a_i}\otimes\ket*{b_j}\bra{a_k}\otimes\bra{b_\ell}
\end{equation*}
In order to get a condition on it's coefficients $p_{ijk\ell}$ we'll take it's trace and force it to be 1.
\begin{align}
\tr\hat{\rho}_{AB} & = \sum_{n, m}\bra{a_n}\otimes\bra{b_m}\qty\Big[\hat{\rho}_{AB}]\ket{a_n}\otimes\ket{b_m} \nonumber \\
& = \sum_{n, m, i, j, k, \ell}p_{ijk\ell}\,\delta_{in}\,\delta_{jm}\,\delta_{kn}\,\delta_{\ell m} \nonumber \\
& = \sum_{i, j}p_{ijij} = 1 \tag{$*$}\label{eq:pcond}
\end{align}
We'll start by computing $\rho_{A}$ in this basis.
\begin{align*}
\tr_B\hat{\rho}_{AB} & = \sum_{n}\1\otimes\bra{b_n}\Big[\hat{\rho}_{AB}\Big]\, \1\otimes\ket{b_n} \\
& = \sum_{n, i, j, k, \ell}p_{ijk\ell}\,\delta_{nj}\,\delta_{\ell n}\ketbra{a_i}{a_k} \\
& = \sum_{n, i, k}p_{inkn}\ketbra{a_i}{a_k}
\end{align*}
Now we can ensure $\tr\hat{\rho}_A = 1$.
\begin{align*}
\tr\hat{\rho}_A & = \sum_\ell \bra{a_\ell}\Big[\hat{\rho}_A\Big]\,\ket{a_\ell} \\
& = \sum_{\ell, n, i, k}p_{inkn}\,\delta_{\ell i}\,\delta_{k\ell} \\
& = \sum_{\ell, n}p_{\ell n \ell n}
\end{align*}
Hence by \cref{eq:pcond} we can conclude $\tr\hat{\rho}_A = 1$.
To show $\hat{\rho}_A$ is positive semi-definite we can use the fact that the sum of the diagonal terms of a density operator are both real and positive. Real because density operators are Hermitian and positive because they are probabilities.
\begin{align*}
\expval{\hat{\rho}_A}{\phi} & = \sum \overline{\phi}_n \bra{a_n}p_{ijkj}\ketbra{a_i}{a_k}\phi_n\ket{a_n} \\
& = \sum\abs{\phi_n}^2 p_{ijkj}\,\delta_{ni}\,\delta_{kn} \\
& = \sum_{ij}\underbrace{\abs{\phi_i}^2}_{\in[0,1]} \underbrace{p_{ijij}}_{\in\R} \\
& \geq \sum_{ij}p_{ijij} = 1 \geq 0
\end{align*}
Hence we've shown the partial trace $\tr_B:\mathrm{L}(H_A\otimes H_B)\to \mathrm{L}(H_A)$ preserves density operators.
\end{solution}
\begin{problem}
Prove that these three pure-state conditions are equivalent.
\end{problem}
\begin{solution}
First we'll show $\mafbox{\hat{\rho}^2 = \hat{\rho} \implies \tr\hat{\rho}^2 = 1}$.
\begin{align*}
\tr\hat{\rho}^2 = \tr\hat{\rho} = 1
\end{align*}
Next we have $\mafbox{\tr\hat{\rho}^2 = 1 \implies \hat{\rho} = \ketbra{\psi}}$.
Let's first calculate $\hat{\rho}^2$ in an othormal basis.
\begin{align*}
\hat{\rho}^2 & = \qty(\sum_{i = 1}^n p_i\ketbra{i}{i})^2 = \sum_{i = 1}^n p_i\ketbra{i}{i}\sum_{j = 1}^n p_j\ketbra{j}{j} \\
& = \sum_{i,j = 1}^n p_ip_j\ket{i}\braket{i}{j}\bra{j} \tag*{$\mafbox{\braket{i}{j} = \delta_{ij}}$} \\
& = \sum_{i = 1}^n p_i^2\ketbra{i}{i}
\end{align*}
Now taking the trace of $\hat{\rho}^2$ we get a condition on the $p_i$'s.
\begin{align*}
\tr \hat{\rho}^2 & = \sum_{k = 1}^n\bra{k}\hat{\rho}^2\ket{k} \\
& = \sum_{k,i = 1}^n\bra{k}\qty(p_i^2\ketbra{i}{i})\ket{k} \\
& = \sum_{i = 1}^n p_i^2 = 1
\end{align*}
So $\sum p_i = 1 = \sum p_i^2$. By basic properties of real numbers, we know $p_i^2 \leq p_i$ when $p_i\in[0, 1]$ and equality only holding when $p_i \in\{0, 1\}$. Using this fact we can write
\begin{equation*}
\sum_{i = 1}^n p_i^2 \leq \sum_{i = 1}^n p_i
\end{equation*}
where again equality only holds if $p_i\in\{0, 1\}$ for all $i$. The only way this can be true is if one of the $p_i$'s is 1 and all of the rest are 0. In that case our summation collapses to one term, and we are left with $\hat{\rho} = \ketbra{i}$ as desired.
Lastly we have $\mafbox{\hat{\rho} = \ketbra{\psi} \implies \hat{\rho}^2 = \hat{\rho}}$.
\begin{align*}
\hat{\rho}^2 = \qty(\ketbra{\psi})\qty(\ketbra{\psi}) = \ket{\psi}\underbrace{\braket{\psi}}_{1}\bra{\psi} = \ketbra{\psi} = \hat{\rho}
\end{align*}
\end{solution}
\problemnumber{5}
\begin{problem}
Prove the existence of the Schmidt decomposition.
\end{problem}
\begin{solution}
Let $\{\ket{a_i}\}$ be an orthonormal basis for $H_A$ and $\{\ket{b_i}\}$ be an orthonormal basis for $H_B$. We then know $\ket{a_n}\otimes \ket{b_m}$ forms a basis for $H_{AB} = H_A\otimes H_B$, and hence we can expand any vector $\ket{\psi}\in H_{AB}$ as
\begin{equation*}
\ket{\psi} = \sum_{ij}\psi_{ij}\ket{a_i}\otimes \ket*{b_j}.
\end{equation*}
We can think of the coefficients $\psi_{ij}$ as a matrix using the association $\ket{a_i}\otimes\ket*{b_j}\cong \ketbra*{a_i}{b_j}$.\footnote{Using the underlying isomorphism of $U^*\otimes V$ and the space $\Hom(U, V)$ of linear maps from $U$ to $V$.} This operator is then taking an element of $H_B$ to an element of $H_A$. That is we can think of the \emph{vector} $\ket{\psi}$ as a linear map $\ket{\psi}_\mathrm{op}: H_B\to H_A$.
With this picture in place we can apply the singular decomposition to write
\begin{equation*}
\ket{\psi}_\mathrm{op} = \sum_{n = 1}^r\lambda_n\ketbra{a_n}{b_n}
\end{equation*}
where $r$ is the rank of $\ket{\psi}_\mathrm{op}$ the \emph{operator}. Now we can map back into $\ket{\psi}$ again using $\ketbra{a_n}{b_m}\cong \ket{a_n}\otimes \ket{b_m}$ and see we have
\begin{equation*}
\ket{\psi} = \sum_{n = 1}^r\lambda_n\ket{a_n}\otimes \ket{b_n}.
\end{equation*}
\end{solution}
\setcounter{section}{3}
\problemnumber{1}
\begin{problem}
Prove the two properties given by Eqns. 3.1.
\end{problem}
\begin{solution}
First we'll show $\sum_\nu E_\nu = \1$.
\begin{align*}
\sum_\nu E_\nu & = \sum_\nu \tr_B\qty(\Pi_\nu\cdot \1_A\otimes \hat{\rho}_B) \\
& = \tr_B\!\qty(\sum_\nu\Pi_\nu\cdot \1_A\otimes\hat{\rho}_B) \\
& = \tr_B\!\qty(\qty[\sum_\nu\Pi_\nu]\cdot \1_A\otimes\hat{\rho}_B) \tag*{$\mafbox{\sum_\nu\Pi_\nu = \1_{A\otimes B}}$} \\
& = \tr_B\!\qty(\1_A\otimes\hat{\rho}_B) \\
& = \1_A
\end{align*}
Now we'll show $E_\nu\geq 0$, but first we need some setup. Since $\Pi_\nu$ is a projector, by the spectral theorem it can be written as
\begin{equation*}
\Pi_\nu = \sum_{ij}\lambda_{ij}\ketbra{a_i b_j}
\end{equation*}
where $\lambda_{ij}$ are real and non-negative. We also have $\hat{\rho}_B = \sum p_n\ketbra{b_n}$ and $\1_A = \sum\ketbra{a_n}$.
\begin{align*}
\Pi_\nu\cdot \1_A\otimes \hat{\rho}_B & = \sum \lambda_{ij} p_n\ket{a_i b_j}\braket{a_i b_j}{a_\ell b_n}\bra{a_\ell b_n} \\
& = \sum\lambda_{ij}p_n\,\delta_{i\ell}\,\delta_{jn}\ketbra{a_i b_j}{a_\ell b_n} \\
& = \sum_{\ell n}\lambda_{\ell n}p_n\ketbra{a_\ell b_n}
\end{align*}
Now we can take the partial trace over $B$.
\begin{align*}
\tr_B\qty(\Pi_\nu\cdot \1_A\otimes\hat{\rho}_B) & = \sum \1_A\otimes\bra{b_i}\lambda_{\ell n}p_n\ketbra{a_\ell b_n}\1\otimes\ket{b_i} \\
& = \sum \lambda_{\ell n}p_n\ketbra{a_\ell}\,\delta_{in}\,\delta_{ni} \\
& = \sum_{\ell n}\lambda_{\ell n}p_n\ketbra{a_\ell}
\end{align*}
Now finally we can can take the expectation values to show $E_\nu$ is positive semi-definite.
I'm a little iffy if this actually works. I feel like there should be a more elegant way to show $E_\nu$ is positive semi-definite, but I can't come up with anything.
\end{solution}
\begin{problem}
Apply Naimark's theorem to identify a \ac{PVM} in an extended Hilbert space that generates the trine.
\end{problem}
\begin{solution}
We'd like to find a set of operators $\{F_i\}$ such that $F_i F_j = F_i \delta_{ij}$ and $\sum_i F_i = \1$ that are built from the operators $E_\nu$. First recall the trine states:
\begin{align*}
\ket{\chi_0} & = \ket{0} & \ket{\chi_1} & = \frac{1}{2}\ket{0} - \frac{\sqrt{3}}{2}\ket{1} & \ket{\chi_2} & = \frac{1}{2}\ket{0} + \frac{\sqrt{3}}{2}\ket{1}
\end{align*}
Now we can calculate the \ac{POVM}s as follows.
\begin{align*}
E_0 & = \frac{2}{3}\ketbra{\chi_0} = \frac{2}{3}\mqty[1 & 0 \\ 0 & 0] \\
E_1 & = \frac{1}{6}\qty(\ketbra{0} - \sqrt{3}\ketbra{0}{1} - \sqrt{3}\ketbra{1}{0} + 3\ketbra{1}) = \frac{1}{6}\mqty[1 & -\sqrt{3} \\ -\sqrt{3} & 3] \\
E_2 & = \frac{1}{6}\qty(\ketbra{0} + \sqrt{3}\ketbra{0}{1} + \sqrt{3}\ketbra{1}{0} + 3\ketbra{1}) = \frac{1}{6}\mqty[1 & \sqrt{3} \\ \sqrt{3} & 3]
\end{align*}
Now we'll look for a $U$ satisfying $U^\dagger\qty(\1\otimes E_\nu)U$. Indeed we can define (by the positive-ness of $E_\nu$)
\begin{equation*}
U = \sqrt{E_0}\otimes\vb{e}_0 + \sqrt{E_1}\otimes\vb{e}_1 + \sqrt{E_2}\otimes\vb{e}_2.
\end{equation*}
Where $\vb{e}_i$ are the standard basis elements of $\C^3$.
This is indeed an isometry ($U^\dagger U = \1$) as
\begin{align*}
U^\dagger U & = \qty(\sqrt{E_0}\otimes\vb{e}_0^\dagger + \sqrt{E_1}\otimes\vb{e}_1^\dagger + \sqrt{E_2}\otimes\vb{e}_2^\dagger)\qty(\sqrt{E_0}\otimes\vb{e}_0 + \sqrt{E_1}\otimes\vb{e}_1 + \sqrt{E_2}\otimes\vb{e}_2) \\
& = E_0\otimes\underbrace{\vb{e}_0^\dagger\vb{e}_0}_{1} + E_1\otimes\vb{e}_1^\dagger\vb{e}_1 + E_2\otimes\vb{e}_2^\dagger\vb{e}_2 \\
& = E_0 + E_1 + E_2 = \1
\end{align*}
Thus we take $F_i = U^\dagger(\1\otimes E_i)U$.
\end{solution}
\begin{problem}
\begin{parts}
\part{Verify that the map $E$ defined in terms of projectors onto coherent states in the example above satisfies the postulates of a \ac{POVM}.}\label{part:33a}
\part{What is the operational interpretation of $\Pr(X) = \tr(E(X)\rho)$ for this \ac{POVM}, noting that $\alpha = \qty(\expval{q}, \expval{p})$ denotes the expectations of the position $q$ and momentum $p$ operatores in the associated coherent state, and that $\Omega = \R^2$ means we are measuring the position and momentum of some particle?}\label{part:33b}
\end{parts}
\end{problem}
\begin{solution}
\ref{part:33a}
To show $E(X)$ defines a valid \ac{POVM} we need to show
\begin{enumerate}
\item $E(X)\geq 0$
\item $E(\R^2) = 1$
\item $E\qty(\bigcup_i X_i) = \sum_i E(X_i)$
\end{enumerate}
\subsubsection*{1}
\begin{equation*}
\expval{E(X)}{\psi} = \frac{1}{\pi}\int_X\dd[2]{\alpha}\braket{\psi}{\alpha}\braket{\alpha}{\psi} = \frac{1}{\pi}\int_X\dd[2]{\alpha}\underbrace{\abs{\braket{\alpha}{\psi}}^2}_{\geq 0} \geq 0
\end{equation*}
This applys for all vectors $\ket{\psi}$, so we conclude $E(X)$ is positive semi-definite.
\subsubsection*{2}
This point is satisfied by the resolution of the identity given in the example prior to the question:
\begin{equation*}
E(\R^2) = \frac{1}{\pi}\int_{\R^2}\dd[2]{\alpha}\ketbra{\alpha} = \1
\end{equation*}
\subsubsection*{3}
Because the $X_i$ are disjoint, the additivity of the Lebesgue integral allow us to split the integral into pieces as follows.
\begin{equation*}
E\qty(\bigcup_i X_i) = \frac{1}{\pi}\int_{\bigcup_i X_i}\dd[2]{\alpha}\ketbra{\alpha} = \sum_i\frac{1}{\pi}\int_{X_i}\dd[2]{\alpha}\ketbra{\alpha} = \sum_{i}E(X_i)
\end{equation*}
The convergence of the (possibly) infinite sum is guaranteed by first using the above manipulation on the first $n$ sets $X_i$ and then taking the limit.
\ref{part:33b}
The operational interpretation of $\Pr(X) = \tr(E(X)\rho)$ is that this is the probability of finding the particle in and maybe around the state space region $X$.
\end{solution}
\end{document}